2
$\begingroup$

When I look at the count of distinct least prime factors for a range of consecutive integers, I am seeing the same minimum number appear again and again. I am wondering if this number represents the true minimum.

Consider a range of consecutive integers defined by $R(x+1,x+c) = x+1, x+2, x+3, \dots, x+c$ with $C(x+1,x+c)$ = the count of distinct least prime factors for $R(x+1,x+c)$

Let $p_k$ be the the $k$th prime which is the greatest prime less than or equal to $\left\lfloor\frac{c}{2}\right\rfloor$

I am finding that for $x \ge 1$, $c \ge 4$, the mimimum $C(x+1,x+c)$ that I can find is $k+2$.

Consider $c=61$, with $\left\lfloor\frac{61}{2}\right\rfloor = 30$ and the greatest prime less than $30$ is $p_{10}=29$. Using the Chinese Remainder Theorem, I can find $x$ where the count of distinct least prime factors is $10+2=12$. In this case, $x=210504408624479$

Here is another example. consider $c=225$, with $\left\lfloor\frac{225}{2}\right\rfloor = 112$ and the greatest prime less than $112$ is $p_{29}=109$. In this case, the minimum that I find is $29+2=31$ with $x=2082073176015230647073633038337038148767197882834487$.

I have a simple java application that allows me to find $x$ for a given $c$ with $k+2$ distinct least prime factors.

Is there a counter example that shows an $x,c$ where $C(x+1,x+c) < k+2$?


Edit: Added $x \ge 1$ and $c \ge 4$ to clarify bounds.

$\endgroup$

1 Answer 1

1
$\begingroup$

I think that the statement "I am finding that for any $x$, any $c$, the mimimum $C(x+1,x+c)$" should be reformulated, clarifying that if we set the value of $x$, then $c$ is free to run on its domain and vice versa, by specifying also which is the aforementioned domain of the pair $(x,c)$ (since the closed interval of positive integers $C$ depends on both $x$ and $c$), otherwise we could simply take $c:=2$ and conclude that it does not exist any pair of consecutive integers containing $k+2$ prime numbers, since $\min(k) : p_k \in \mathbb{P} = 1$ and then $\min_k(p_k)+2 \geq 3$ if $k \in \mathbb{Z}^+$ by hypothesis.

Basically, I have constructed my counterexample here by simply taking $\overline{c} := 2$ so that $\left\lfloor\frac{\overline{c}}{2}\right\rfloor = \left\lfloor\frac{2}{2}\right\rfloor = 1$ and it follows that $[x+1, x+2]$ cannot contain more primes than the cardinality of the set {x+1, x+2} itself (i.e., two), but it is trivial to point out that $\nexists p_k \in \mathbb{P} : p_k \leq \left\lfloor\frac{\overline{c}}{2}\right\rfloor$, while for $c:=4$ we can find $k+2=3$ primes if $x=1$ (i.e., by considering the peculiar set {2, 3, 4, 5} such that $p_k=p_1=2$ is (least or) equal to $\left\lfloor\frac{4}{2}\right\rfloor$).

Anyway, I am starting to think about Rosser's Theorem (or its further improvements as Dusart's bound), maybe it would just be enough to give you a proper answer (since $p_k > k \cdot k(\log(k))$ holds for any $k$ as above).

$\endgroup$
4
  • $\begingroup$ Yeah, my only two concerns arose from the proper setting of the $c$ domain (which was unspecified and $c \in \mathbb{Z}^+$ doesn't work if we assume that $p_k$ has to exist by hypothesis... on the other hand, as I wrote, $c \geq 4$ is perfectly fine, IMHO, since $2$ is a very special prime), whereas my second concern was about the opportunity to clarify better in the text (I know, it is a trivial thing) that we cannot set both $c$ and $x$ at the same time. $\endgroup$ Commented May 1, 2023 at 1:38
  • $\begingroup$ I have updated the bounds. For $𝑐=4, 𝑝_1=2=\left\lfloor\frac{4}{2}\right\rfloor$ so that $𝑘=1$ and the minimum expected is $1+2=3$ distinct least prime factors which matches $\{2,3,5\}$ in your example. Am I misunderstanding your point? $\endgroup$ Commented May 1, 2023 at 1:38
  • 2
    $\begingroup$ Thank for explaining. I agree. I should have added the bounds in the original question. :-) $\endgroup$ Commented May 1, 2023 at 1:39
  • $\begingroup$ You are welcome :-) $\endgroup$ Commented May 1, 2023 at 1:40

You must log in to answer this question.

Start asking to get answers

Find the answer to your question by asking.

Ask question

Explore related questions

See similar questions with these tags.